LSAT and Law School Admissions Forum

Get expert LSAT preparation and law school admissions advice from PowerScore Test Preparation.

 Administrator
PowerScore Staff
  • PowerScore Staff
  • Posts: 8919
  • Joined: Feb 02, 2011
|
#40232
Complete Question Explanation
(The complete setup for this game can be found here: lsat/viewtopic.php?t=8561)

The correct answer choice is (D)

The question asks us to determine a pair of buildings that cannot both have been owned by the Trents. Since each answer choice contains a only two buildings, the 2-2-1 distribution applies. Since the Templates represent an exhaustive range of solutions, eliminating the four answer choices that could be true should be a relatively simple task. If you decided against the use of templates, you would need to identify an answer choice that violates at least one of the three rules, which would be significantly more time-consuming.

Answer choice (A): This answer choice is incorrect, because the Trents could have owned both F and G without violating any of the rules, as shown in Template 3A.

Answer choice (B): This answer choice is incorrect, because the Trents could have owned both G and M without violating any of the rules, as shown in Template 3A.

Answer choice (C): This answer choice is incorrect, because the Trents could have owned both G and S without violating any of the rules, as shown in Template 2A.

Answer choice (D): This is the correct answer choice, as none of the 2-2-1 templates allow the Trents to have owned both I and M. Here’s why:

If the Trents owned both I and M, and two is the maximum number of buildings that they could have owned, then the Trents cannot have owned the stable. To comply with the third rule, the Yandells must have owned I, which cannot be true since the Trents own I. Answer choice (D) cannot be true, because it violates the last rule of the game.

Answer choice (E): This answer choice is incorrect, because the Trents could have owned both I and S without violating any of the rules, as shown in Template 1A.
 quan-tang@hotmail.com
  • Posts: 35
  • Joined: Sep 18, 2022
|
#98853
do you recommend us to skip question like this? I skipped the question because this questions templates is time consuming complete, and all questions except this one can be answered without templates
 Adam Tyson
PowerScore Staff
  • PowerScore Staff
  • Posts: 5153
  • Joined: Apr 14, 2011
|
#99245
If you did not choose to do templates at the beginning, then this question might be better done at the end of the game after you have created some local diagrams that you can use to help eliminate losers and limit how many answer choices you have to test.

Get the most out of your LSAT Prep Plus subscription.

Analyze and track your performance with our Testing and Analytics Package.